4th order Maclaurin Polynomial - Help with error approximation

Click For Summary
The discussion focuses on finding the upper bound for the error in a 3rd order Maclaurin polynomial approximation of e^(3x) at x = 1/3. Initially, the user calculated the error bounds using the 4th derivative, which they later corrected from 27e^(3z) to 81e^(3z). After recalculating, the new error bounds were approximately 0.04167 and 0.11326, indicating a significant reduction in the estimated error. The user also corrected their understanding of the difference between the actual value of e and the approximation, realizing it was approximately 0.05 rather than 0.5. The thread highlights the importance of careful derivative calculations in error approximation.
opticaltempest
Messages
135
Reaction score
0
I have the the follow 3rd order polynomial approximation for e^3x

<br /> f(x) = e^{3x} \approx 1 + 3x + \frac{9}{2}x^2 + \frac{9}{2}x^3 <br />

In an earlier part of the problem, I found

<br /> f\left( {\frac{1}{3}} \right) = e^{3\left( {\frac{1}{3}} \right)} <br />

<br /> \approx 1 + 3\left( {\frac{1}{3}} \right) + \frac{9}{2}\left( {\frac{1}{3}} \right)^2 + \frac{9}{2}\left( {\frac{1}{3}} \right)^3 = \frac{8}{3} \approx 2.667<br />

I need to find the upper bound for the error of the above approximation.

<br /> f(x) = e^{3x} = 1 + 3x + \frac{9}{2}x^2 + \frac{9}{2}x^3 + R_3 (x)<br />

<br /> \begin{array}{l}<br /> R_3 \left( {\frac{1}{3}} \right) = \frac{{f^{\left( 4 \right)} \left( z \right)}}{{4!}}\left( {\frac{1}{3}} \right)^4 \\ <br /> {\rm{where }}0 &lt; z &lt; \frac{1}{3} \\ <br /> \end{array}<br />

<br /> f^{\left( 4 \right)} (z) = 27e^{3z} <br />

So the bounds on my error

<br /> \frac{{27e^{3(0)} }}{{4!}} &lt; \frac{{27e^{3z} }}{{4!}} &lt; \frac{{27e^{3\left( {\frac{1}{3}} \right)} }}{{4!}}<br />

Which means my error for this function when evaluated at 1/3 is
somewhere between 1.125 and ~3.058. This must be wrong
because f(1/3) is approx. 2.67.
(e - 2.67) is approx 0.5.

I believe the bounds that I found are saying that my error must be
greater than 1.125 and less than 3.058. Where did I go wrong?

Thanks

EDIT: I found some errors while looking over the problem more carefully.
Please let me know if it looks correct now. Thanks. My corrections are
below.

<br /> f^{\left( 4 \right)} (z) = 81e^{3z} <br />

<br /> \frac{{81e^{3(0)} }}{{4!}}\left( {\frac{1}{3}} \right)^4 &lt; \frac{{81e^{3(z)} }}{{4!}}\left( {\frac{1}{3}} \right)^4 &lt; \frac{{81e^{3\left( {\frac{1}{3}} \right)} }}{{4!}}\left( {\frac{1}{3}} \right)^4 <br />

<br /> \approx 0.04167 &lt; \frac{{81e^{3(z)} }}{{4!}}\left( {\frac{1}{3}} \right)^4 &lt; \approx 0.11326<br />
 
Last edited:
Physics news on Phys.org
(e - 2.67) is approx 0.5.
This still looks wrong.
 
Oops,

(e - 2.67) is approx. 0.05 not 0.5
 
Question: A clock's minute hand has length 4 and its hour hand has length 3. What is the distance between the tips at the moment when it is increasing most rapidly?(Putnam Exam Question) Answer: Making assumption that both the hands moves at constant angular velocities, the answer is ## \sqrt{7} .## But don't you think this assumption is somewhat doubtful and wrong?

Similar threads

Replies
4
Views
2K
Replies
6
Views
3K
  • · Replies 7 ·
Replies
7
Views
2K
  • · Replies 105 ·
4
Replies
105
Views
6K
  • · Replies 5 ·
Replies
5
Views
2K
Replies
4
Views
2K
  • · Replies 7 ·
Replies
7
Views
2K
  • · Replies 9 ·
Replies
9
Views
2K
  • · Replies 5 ·
Replies
5
Views
1K
  • · Replies 4 ·
Replies
4
Views
1K